Dot Product of a Unit Vector with the Negative of itself

Click For Summary
The discussion revolves around calculating the flux for the octant of a sphere using dot products and the differential area element (dA). The user has already completed calculations for specific quadrants and is questioning how the negative unit vector affects the integral when evaluating the flux. They assert that the flux will be zero when the angle is zero, but seek clarification on whether the negative in the dot product will cancel out. The user proposes testing this by explicitly calculating the dot product of a unit vector with its negative. The conversation emphasizes understanding the implications of negative unit vectors in flux calculations.
EarthDecon
Messages
8
Reaction score
0

Homework Statement



I am trying to calculate the flux for the octant of a sphere, and I am trying to figure out how the mathematics, dot products, and dA works in the integral. I already did the quadrant for \hat{θ} where θ= π/2 (the bottom quadrant) and I did the left quadrant where \hat{n} = -\hat{ø} but I want to understand, when you are calculating \int_{S}\vec{F}\bullet\hat{ø}dA , for F_{ø}, if ø is negative for the left quadrant, I know that the flux will be 0 because ø = 0 and sinø = 0 but in the integral, will the negative cancel out?

Homework Equations


Here is a picture for the octant of the sphere, my question is on the left quadrant: http://2.bp.blogspot.com/_N2c1FjhWLag/TF_yaJpc6DI/AAAAAAAAACI/4hhPUla483A/s1600/sphel.gif

To solve for the total flux we have the equation:
\oint \vec{F}\bullet\hat{n}dA = \int_{S1}\vec{F}\bullet\hat{n}dA + \int_{S2}\vec{F}\bullet\hat{n}dA + \int_{S3}\vec{F}\bullet\hat{n}dA + \int_{S4}\vec{F}\bullet\hat{n}dA

\vec{F} (r,θ,ø) = (r^{2}cosθ)\hat{r}+(r^{2}cosø)\hat{θ}-(r^{2}cosθsinø)\hat{ø}


The Attempt at a Solution


I already know that the flux is equal to zero as I said, I just wanted to be sure that the dot product of two unit vectors in this case is -1 or if there's more calculation needed for this.
 
Physics news on Phys.org
Why not check by explicitly taking a unit vector ##\hat u = (1,0)## and formally constructing the dot product of that with the negative of itself?
 
Question: A clock's minute hand has length 4 and its hour hand has length 3. What is the distance between the tips at the moment when it is increasing most rapidly?(Putnam Exam Question) Answer: Making assumption that both the hands moves at constant angular velocities, the answer is ## \sqrt{7} .## But don't you think this assumption is somewhat doubtful and wrong?

Similar threads

  • · Replies 9 ·
Replies
9
Views
1K
  • · Replies 9 ·
Replies
9
Views
2K
  • · Replies 13 ·
Replies
13
Views
2K
  • · Replies 9 ·
Replies
9
Views
2K
  • · Replies 4 ·
Replies
4
Views
2K
  • · Replies 16 ·
Replies
16
Views
3K
Replies
20
Views
2K
  • · Replies 3 ·
Replies
3
Views
2K
Replies
3
Views
2K
  • · Replies 1 ·
Replies
1
Views
3K